someone help me please with this algebra problem

Someone Help Me Please With This Algebra Problem

Answers

Answer 1

Answer:

x=44-y

Step-by-step explanation:

In this problem, there are 44 glasses in total. Additionally, we want to know how many ice teas she can have; therefore, we must find x. The problem tells us that when x and y are added together they should equal 44. One way to solve this is to write the equation, x+y=44, because we know it is true and then solve for x. To find x subtract y from both sides. This equals x=44-y, the final answer.

Answer 2
x=44-y
explanation: y is the lemonade, and every time a glass of it is poured it is taking away from the 44 glasses. after the lemonade is poured it leaves x iced tea glasses. you can eliminate half of the answers off the bat because you know you’re looking for iced tea glasses, so whatever equation is there must equal x which elimates option 1 and 3.

Related Questions

I would raise it to 1grand but I’m very broke

Answers

Answer:

ITS A LOL

Step-by-step explanation:

If
5
3 cosα = in the first quadrant, what does cot α
equal?

Answers

Answer:

5/4

Step-by-step explanation:

To Find :-

cot a .

Solution :-

By question ,

=> cos a = 5/3 = b/h

=> p = √ 5² - 3² = √ 25 -9 = 4

Therefore ,

=> cot a = b/p = 5/4

In which section of the number line is 32−−√?

Answers

where's the number line?

maybe u can attach it at the comments:)

Answer:

Section B

Step-by-step explanation:

What is 6.273 rounded to the nearest thousandths?

Answers

Answer:

6.270

Step-by-step explanation:

3 is below 5 so you just turn it into a 0. If it was 5 or above you would just add a number to the 7.

The polygons are similar, but not necessarily drawn to scale. Find the value of x. PLEASE HELPPPP

Answers

Answer:

x = 27.5.

Step-by-step explanation:

There are given numbers on each side. If the figures are similar, then they have a set ratio for each value.

So, 55:8 and x:4. If you want to, you can flip it, so that it is 8:55 and 4:x.

With that in mind, it is easy to see what the ratio is. Because 4 is half of 8, x is half of 55. 55 divided by 2 is 27.5.

Therefore, x = 27.5.

Find the value of y

Help please

Answers

Answer:

6

Step-by-step explanation:

Set your formula up as

15 = 2y+3

15 - 3 = 2y

12 = 2y

12 / 2 = y

6 = y

The equation d = m/v can be used to calculate the density,d, of an object with mass,m, and volume,V. Which is an equivalent equation solver for V?

Answers

Answer:

v = m/d

Step-by-step explanation:

since

[tex]d = \frac{m}{v} \: \\ \\ v = \frac{m}{d} [/tex]

If f(x) = 2x2 + 1, what is f(x) when x = 3?

Answers

Answer:

18

Step-by-step explanation:

f(x)=2x(2+1)

f(3)=2(3)(2+1)

    =6(2+1)

    =12+6

    =18

The sum of 9 and a number is less than 12. Which of the following
inequalities expresses the solution?

Answers

Answer:

9+x<12

Step-by-step explanation:

add 9 to an unknown number.

9+x

use the < less than symbol (instead of an equal sign) and write 12.

Answer:

x + 9  < 12

Step-by-step explanation:

Let the unknown number = x

Sum of 9  and x  :  x + 9

x + 9  < 12


Find (a) the mean, (b) the median, and
(c) the mode of the number of verses in
the first eight chapters of Proverbs.
Proverbs 1 23 4 5 6 7 8
Verses 33 22352723352736
(a)
(b)
(c)

Answers

Answer:

Mean = 33+ 22+35+27+23+35+27+36 = 238/8 = 29.75

Median = 27+23 = 50/2 = 25

Mode = 35

Range = 36 - 22 = 14

Step-by-step explanation:

Stay Blessed

determine if 5yx - 17xy are like terms​

Answers

Step-by-step explanation:

yes they are because they have the same variables which are X& Y

Answer:

5yx-7xy

they are like terms, it's all multiplication just a different arrangement

5xy-7xy=-2xy

Step-by-step explanation:

hope this is helpful

find the volume of the rectangular prism. plz answer this lol

Answers

Answer:

.....how when the dimensions are not even clear lol

Answer:

48 cm³

Step-by-step explanation:

the volume of a rectangular prism= length × breadth × height

= 8× 3 × 2

= 48 cm³

a 10 foot ladder rests against a vertical wall if the bottom of the ladder slides away from the wall at a speed of 2 ft/s how fast is the angle betwween the top of the ladder and the wall changing when that angle is

Answers

Answer:

d∅/dt = √2/5 Rad/sec

Step-by-step explanation:

According to the Question,

Given That, a 10-foot ladder rests against a vertical wall if the bottom of the ladder slides away from the wall at a speed of 2 ft/s how fast is the angle between the top of the ladder and the wall changing when that angle is π/4.

Solution,

Let x be the Distance between the base of the wall and the bottom of the ladder.

and let ∅ be the angle between the top of the ladder and the wall.

Then, Sin∅ =x/10  so, x=sin∅ *10

Differentiating with respect to time t we get,

dx/dt = 10 * cos∅ * d∅ /dt

We have given that dx/dt = 2 ft/s and ∅ =π/4

Now, Put these value we get

2 = 10 *(cos(π/4))* d∅/dt

2 = 10/√2 * d∅/dt

d∅/dt = √2/5 Rad/sec

solve the simultaneous equation: x-y=2
xy=36​

Answers

Answer:

Y= –7.08, 5.08

Step-by-step explanation:

hope ya ready bro.

X=36/Y

replace 36/y instead of X

36/Y–Y=2===> 36–Y²=2Y===> Y²+2Y–36=0

Y1= –1+√37≈ –7.08

Y2= –1–√37≈ 5.08

Students were asked to choose their favorite season. If 2/9 of the students
chose spring and 5/9 chose summer, what fraction chose either spring or
summer?

Answers

Answer:

7/9

Step-by-step explanation:

2/9+5/9=7/9

Arvin has $10000 to invest. He invests part in a term deposit paying 5%/year, and the remainder in Canada savings bonds paying 3.4%/year. At the end of the year, he earned simpler interest of $413. How much did he invest at 5%/year?

Answers

Answer:

$4,562.5

Step-by-step explanation:

The amount Arvin has to invest, P = $10,000

The interest paid on the investment in the term deposit = 5%/year

The interest paid om the investment in Canada savings bonds = 3.4%/year

The amount Arvin earned at the of the year as simple interest, A = $413

Let, x, represent the amount Arvin invested in the term deposit and let, y, represent the amount he invested in Canada savings bonds, we can get the following system of equations

x + y = 10,000...(1)

0.05·x + 0.034·y = 413...(2)

Making y the subject of equation (1) and substituting the value in equation (2), we get;

From equation (1), we get, y = 10,000 - x

Plugging the above value of y in equation (2) gives;

0.05·x + 0.034 × (10,000 - x) = 413

∴ 0.05·x - 0.034·x + 340 = 413

x = (413 - 340)/(0.05 - 0.034) = 4,562.5

Therefore, the amount Arvin invested in the term deposit at 5%, x = $4,562.5

(y = 10,000 - x

∴ y = 10,000 - 4,562.5 = 5,437.5

The amount Arvin invested in Canada savings bonds, y = $5,437.5.)

if u help me i will give you brainliest <3

Answers

Answer:

a) 6

b) 12:35 ( not so sure about this one)

Step-by-step explanation:

a)

distance = speed * time

ruths distance = 4* 1.5 (because it takes her 1h 30 mins)

distance = 6miles    

b)  1/4 here is quarter so, 15mins

10:30 and then add 50 mins to that.

     she leaves at 11:20 and takes 1h and 15 mins ( 75mins) to walk home

     so 11:20 + 1h 15min

     = 12:35

hope this helps :)

Answer:

a) 6 miles

b) 12:35 pm

Step-by-step explanation:

a) 9 am to 10:30 am = 1 hour and 30 mins = 1.5 hours

4mph x 1.5 = 6miles

b) 10:30 am + 50 mins = 11:20 am

1 hour = 60 mins

60/4 = 15 mins

1 1/4 = 1 hour 15 mins

11:20 am + 1 hour 15 mins = 12:35 pm

a^2×c^2/c^2×d^2+bc/ad reduce the algebraic ​

Answers

Answer:

[tex]\frac{a^2*c^2}{c^2*d^2}+\frac{bc}{ad}= \frac{a^3 + bcd}{ad^2}[/tex]

Step-by-step explanation:

Given

[tex]\frac{a^2*c^2}{c^2*d^2}+\frac{bc}{ad}[/tex]

Required

Simplify

We have:

[tex]\frac{a^2*c^2}{c^2*d^2}+\frac{bc}{ad}[/tex]

Cancel out [tex]c^2[/tex]

[tex]\frac{a^2*c^2}{c^2*d^2}+\frac{bc}{ad}= \frac{a^2}{d^2}+\frac{bc}{ad}[/tex]

Take LCM

[tex]\frac{a^2*c^2}{c^2*d^2}+\frac{bc}{ad}= \frac{a^3 + bcd}{ad^2}[/tex]

The sum of three consecutive odd numbers is 63. What are the numbers?​

Answers

19,21,23. Adding all of these consecutive odd numbers is equal to 63

Answer:

19, 21 and 23

Step-by-step explanation:

→ Make an algebraic expression for the 3 consecutive numbers

2x + 1, 2x + 3 and 2x + 5

→ Add the expressions together and make it equal to 63

2x + 1 + 2x + 3 + 2x + 5 = 63

→ Simplify

6x + 9 = 63

→ Minus 9 from both sides

6x = 54

→ Divide both sides by 6

x = 9

→ Resubstitute x = 9 into the 3 expressions

x = 9 into 2x + 1 is 19

x = 9 into 2x + 3 is 21

x = 9 into 2x + 5 is 23

plz help me with this math and also explain

Answers

Step-by-step explanation:

[1]

SI = $250Rate (R) = 12[tex] \sf \dfrac{1}{2}[/tex] %Time (t) = 4 years

[tex]\longrightarrow \tt { SI = \dfrac{PRT}{100} } \\ [/tex]

[tex]\longrightarrow \tt { 250 = \dfrac{P \times 12\cfrac{1}{2} \times 4}{100} } \\ [/tex]

[tex]\longrightarrow \tt { 250 = \dfrac{P \times \cfrac{25}{2} \times 4}{100} } \\ [/tex]

[tex]\longrightarrow \tt { 250 = \dfrac{P \times 25 \times 2}{100} } \\ [/tex]

[tex]\longrightarrow \tt { 250 = \dfrac{P \times 50}{100} } \\ [/tex]

[tex]\longrightarrow \tt { 250 \times 100 = P \times 50} \\ [/tex]

[tex]\longrightarrow \tt { 25000 = P \times 50} \\ [/tex]

[tex]\longrightarrow \tt { \dfrac{25000}{50} = P } \\ [/tex]

[tex]\longrightarrow \underline{\boxed{ \green{ \tt { \$ \; 500 = P }}}} \\ [/tex]

Therefore principal is $500.

__________________

[2]

2/7 of the balls are red.3/5 of the balls are blue.Rest are yellow.Number of yellow balls = 36

Let the total number of balls be x.

→ Red balls + Blue balls + Yellow balls = Total number of balls

[tex]\longrightarrow \tt{ \dfrac{2}{7}x + \dfrac{3}{5}x + 36 = x} \\ [/tex]

[tex]\longrightarrow \tt{ \dfrac{10x + 21x + 1260}{35} = x} \\ [/tex]

[tex]\longrightarrow \tt{ \dfrac{31x + 1260}{35} = x} \\ [/tex]

[tex]\longrightarrow \tt{ 31x + 1260= 35x} \\ [/tex]

[tex]\longrightarrow \tt{ 1260= 35x-31x} \\ [/tex]

[tex]\longrightarrow \tt{ 1260= 4x} \\ [/tex]

[tex]\longrightarrow \tt{ \dfrac{1260 }{4}= x} \\ [/tex]

[tex]\longrightarrow \underline{\boxed{ \tt { 315 = x }}} \\ [/tex]

Total number of balls is 315.

A/Q,

3/5 of the balls are blue.

[tex]\longrightarrow \tt{ Balls_{(Blue)} =\dfrac{3 }{5}x} \\ [/tex]

[tex]\longrightarrow \tt{ Balls_{(Blue)} =\dfrac{3 }{5}(315)} \\ [/tex]

[tex]\longrightarrow \tt{ Balls_{(Blue)} = 3(63)} \\ [/tex]

[tex]\longrightarrow \underline{\boxed{ \green {\tt { Balls_{(Blue)} = 189 }}}} \\ [/tex]

Need help ASAP !!!when creating a question for a survey , the way a question is written:

Answers

Answer:

The third one i think

Step-by-step explanation:

Since a survey shouldn't be biased or anything.

A ABC has vertices A (-7, -13), B (12, -8), and C(-17, 19). Which of the following represents the
reflection of AABC across the line y = x and its rotation of 90° about the origin?

Answers

Answer:

brainliest and 20 points! Which best explains whether or not ΔABC ≅ ΔLMN? The figures are congruent because a 270° rotation about the origin and then a reflection over the x-axis will map ΔABC onto ΔLMN. The figures are congruent because a 90 rotation about the origin and then a reflection over the x-axis will map ΔABC onto ΔLMN. The figures are not congruent because point B corresponds with point N and point C corresponds with point M. The figures are not congruent because there is no rigid transformation or combination of rigid transformations that will map ΔABC onto ΔLMN.

Step-by-step explanation:

it's is the answr

A lending library has a fixed charge for the first three days and an additional charge for each day thereafter. Saritha paid ₹27 for a book kept for seven days, while Susy paid ₹21 for the book she kept for five days. Find the fixed charge and the charge for each extra day.​​

Answers

Answer:

Ji Gud afternoon.

I'm fine

what about you madam.

Hope u r fine and stay safe and healthy


[tex]solve : - \\ \\ (4 {}^{2} + 5 {}^{2} ) = {?}[/tex]

Answers

Step-by-step explanation:

4² = 16

5² = 25

16+25 = 41

41 is the answer.

Hope it helps! :)

Answer:

[tex]( {4}^{2} + {5}^{2} ) \\ (16 + 25) \\ = 41[/tex]

Draw graph for equation 0.25x+0.50y<3

Answers

that would be the graph for your equation :)

Does anyone know the answer to this question?

Answers

The answer is A; g(x) = |x+4| - 2

help help help pls :)

Answers

Answer:

[tex]opposite\approx 70.02[/tex]

Step-by-step explanation:

The triangle in the given problem is a right triangle, as the tower forms a right angle with the ground. This means that one can use the right angle trigonometric ratios to solve this problem. The right angle trigonometric ratios are as follows;

[tex]sin(\theta)=\frac{opposite}{hypotenuse}\\\\cos(\theta)=\frac{adjacent}{hypotenuse}\\\\tan(\theta)=\frac{opposite}{adjacent}[/tex]

Please note that the names ([tex]opposite[/tex]) and ([tex]adjacent[/tex]) are subjective and change depending on the angle one uses in the ratio. However the name ([tex]hypotenuse[/tex]) refers to the side opposite the right angle, and thus it doesn't change depending on the reference angle.

In this problem, one is given an angle with the measure of (35) degrees, and the length of the side adjacent to this angle. One is asked to find the length of the side opposite the (35) degree angle. To achieve this, one can use the tangent ([tex]tan[/tex]) ratio.

[tex]tan(\theta)=\frac{opposite}{adjacent}[/tex]

Substitute,

[tex]tan(35)=\frac{opposite}{100}[/tex]

Inverse operations,

[tex]tan(35)=\frac{opposite}{100}[/tex]

[tex]100(tan(35))=opposite[/tex]

Simplify,

[tex]100(tan(35))=opposite[/tex]

[tex]70.02\approx opposite[/tex]

Solve for x. Round to the nearest tenth, if necessary.

Answers

Answer:

25.5402665394

Step-by-step explanation:

x=24/cos(20) = 25.54026653942

The measure of an angle is eight times the measure of its supplementary angle. What is the measure of each angle?

Answers

Answer:

160°

Step-by-step explanation:

Let one angle be x

Let second angle be 8x

Sum of supplementary angles = 180°

x+ 8x = 180°

9x = 180

x = 180/9 = 20

First angle= x= 20°

Second angle = 8x = 8*20 = 160°

what is the value of b in this equation?

A. -20
B. -6
C. 6
D. 20

Answers

Answer:

Step-by-step explanation:

a- 12

b- 2

c- 11

and the next part is

x- 6

y- 18

Answer:

B

Step-by-step explanation:

Using the rules of exponents

[tex](a^m)^{n}[/tex] = [tex]a^{mn}[/tex]

[tex]a^{-m}[/tex] = [tex]\frac{1}{m}[/tex]

Then

[tex](y^b)^{4}[/tex] = [tex]\frac{1}{y^{24} }[/tex]

[tex]y^{4b}[/tex] = [tex]y^{-24}[/tex]

Equate the exponents

4b = - 24 ( divide both sides by 4 )

b = - 6 → C

Other Questions
Can we update App Store in any apple device. (because my device is kinda old and if want to download the recent apps it aint showing them). So is it possible to update???Please help Kim lives in Maine. She is 17 years old and was born in Canada. Which statement about her US citizenship status is correct? A. Kim has already become a naturalized US citizen. B. Because Kim lives in the United States, she is a citizen. C. Kim will never be able to be a US citizen because she was born in Canada. D. Kim cannot apply to become a naturalized citizen until she turns 18. Which lines from DH Lawerence's "Piccadilly Circus at Night" contain a metaphor? A. Our faces flower for a liste hour. Daisies that walan ailen B. When into the night the yellow light is roused like dust above the C. Or like a mist the moon has issed from off a pool in the midst of the downs A rock group assigns its payment under a performance contract to the Costume Shop, a business that has supplied the group with outrageous stage outfits, and to which the group owes a great sum of money. Under this arrangement the rock group is the a. delegator. b. delegatee. c. assignee. d. assignor. Which of the following is a correct tangent ratio of the figure?tan 72= 58/19tan 19 =58/72tan 58 = 72/19tan 72 = 19/52 At a hockey game, a vender sold a combined total of 228 sodas and hot dogs. The number of sodas sold was two times the number of hot dogs sold. Find the number of sodas and the number of hot dogs sold. The Great Gatsby: Chapters 6-7 quizWhen they come over on horseback to Gatsbys house, what do Mr. and Mrs. Sloane and Tom do to Gatsby thats extremely rude? apakah tugas golongan brahmin dalam kerajaan alam melayu? Find the value of t for a t-distribution with 12 degrees of freedom such that the area to the left of t equals 0.025. Round your answer to three decimal places, if necessary. Which of the following characteristics must an object possess in order to be considered alive? Select all that apply Olin is a partner in Precision Plumbing. When the partners decide to dissolve the firm, Olin collects and distributes the assets. This results in Which Company/Security report would be best for someone looking to compare a company to its peers in a single display What is potential energy? What are some of its examples. Question 4 of 10Which of the following passages from "The Jilting of Granny Weatherall"shows imagery?O A. Nonsense. It was strange about children. They disputed your everyword.O B. She tried to remember. No, I swear he never harmed me but inthat.C. She was strong, in three days she would be as well as ever.O D. Dark colors with the shadows rising towards the ceiling in longangles. A manufacturer inspects a sample of 500 smart phones and finds that 496 of them have no defects. The manufacturer sent a shipment of 2000 smartphones to a distributor. Predict the number of smartphones in the shipment that are likely to have no dects. We the people of the United States in order to form a more perfect union established justice ensure domestic tranquility provide for the common defense promote the general welfare and secure the blessings of liberty to ourselves and prosperity do ordain and stylish this Constitution for the United States of America which idea is expressed in this expert from the constitution Using complete sentences, explain the difference between an indirect democracy and a direct democracy Name two of the organizations that are involved in standards or installation requirements for home automation systems: Find the vertex of f(x) = 3(x-7)(x+5)Please show your work. ^^ Ive been having a bit of trouble with this question :( although its simple quadratics is not my strong suit Using the map, what fact canbe made about the relocationof Japanese-American duringWorld War II?A. Most of the camps can be found in themidwestB. Most camps can be found in theWestern interior of the United StatesC. California was the only state that hadcamps